Last visit was: 26 Apr 2024, 23:12 It is currently 26 Apr 2024, 23:12

Close
GMAT Club Daily Prep
Thank you for using the timer - this advanced tool can estimate your performance and suggest more practice questions. We have subscribed you to Daily Prep Questions via email.

Customized
for You

we will pick new questions that match your level based on your Timer History

Track
Your Progress

every week, we’ll send you an estimated GMAT score based on your performance

Practice
Pays

we will pick new questions that match your level based on your Timer History
Not interested in getting valuable practice questions and articles delivered to your email? No problem, unsubscribe here.
Close
Request Expert Reply
Confirm Cancel
SORT BY:
Date
Math Expert
Joined: 02 Sep 2009
Posts: 92948
Own Kudos [?]: 619287 [3]
Given Kudos: 81609
Send PM
GMAT Club Legend
GMAT Club Legend
Joined: 12 Sep 2015
Posts: 6818
Own Kudos [?]: 29944 [4]
Given Kudos: 799
Location: Canada
Send PM
GMAT Club Legend
GMAT Club Legend
Joined: 18 Aug 2017
Status:You learn more from failure than from success.
Posts: 8020
Own Kudos [?]: 4098 [0]
Given Kudos: 242
Location: India
Concentration: Sustainability, Marketing
GMAT Focus 1:
545 Q79 V79 DI73
GPA: 4
WE:Marketing (Energy and Utilities)
Send PM
Target Test Prep Representative
Joined: 14 Oct 2015
Status:Founder & CEO
Affiliations: Target Test Prep
Posts: 18767
Own Kudos [?]: 22065 [0]
Given Kudos: 283
Location: United States (CA)
Send PM
Re: Let S be a set of 6 integers taken from {1, 2, ..., 12} with the prope [#permalink]
Expert Reply
Bunuel wrote:
Let S be a set of 6 integers taken from {1, 2, ..., 12} with the property that if a and b are elements of S with a < b, then b is not a multiple of a. What is the least possible value of an element in S?

A. 2
B. 3
C. 4
D. 5
E. 7


Let’s start with 2 as the smallest number, as 1 is not among the answer choices. We can have 2, 3, 5, 7, 11 in our set. However, since any other number we will add will either be a multiple of 2 or a multiple of 3, this set of numbers does not work. Also, there are no numbers in the set which will enable us to add more than one number if we don’t include (for instance, if we don’t add 3, we will be able to add 9 but we still have five numbers. If we don’t include 5, we will be able to add 10 but we still have five numbers).

Now, let’s try 3 as the smallest number. We can have 3, 4, 5, 7, 11. However, again any other number we add will be a multiple of a number that is already in the set (6 is a multiple of 3, 8 is a multiple of 4, 10 is a multiple of 5 etc.) Again there are no numbers which would make it possible to add more than one number if not included, similar to the above case.

Let’s try 4 as the smallest number. We can have 4, 5, 6, 7, 9, 11. We see that we have followed the rules for the set, so 4 is the smallest possible number.

Answer: C
GMAT Club Bot
Re: Let S be a set of 6 integers taken from {1, 2, ..., 12} with the prope [#permalink]
Moderators:
Math Expert
92948 posts
Senior Moderator - Masters Forum
3137 posts

Powered by phpBB © phpBB Group | Emoji artwork provided by EmojiOne